what is sepsis? - bsu.edu.eg and mod.docx  · web viewthe word is used in this module in a looser...

Post on 03-Jan-2019

212 Views

Category:

Documents

0 Downloads

Preview:

Click to see full reader

TRANSCRIPT

INTRODUCTION

Intensive care units were established to provide support of vital organ function during a time of life-threatening physiologic dysfunction. Their success is reflected in the fact that the vast majority of patients admitted to an ICU survive an illness that half a century ago would have been rapidly lethal. However, in the successes of intensive care lie the roots of its greatest unsolved challenges.

Sepsis is a lethal disease process, with mortality rates of 25% to 60% or higher, depending on the severity of the illness, but also varying substantially from one country to the next. By convention, the word sepsis is used to describe the syndrome of systemic inflammation when infection is the cause, while the systemic inflammatory response syndrome or SIRS denotes that process without reference to its causative trigger. However, it may be difficult to determine whether infection is truly present, and if it is, whether it is the cause, or the consequence of the deterioration that is evolving. The word is used in this module in a looser clinical sense, to denote a syndrome that is commonly, but not always caused by infection, but that heralds significant risk to the patient, and tremendous challenges to the clinician. Nevertheless, when sepsis is suspected it is crucial to look for infection, as the rapid and effective treatment of infection improves outcome.

Multiple organ dysfunction syndrome or MODS is responsible for 80% of all ICU deaths, and evolves in the wake of a syndrome of systemic inflammation. Sometimes inflammation is a consequence of a sterile insult such as pancreatitis, but more often, it is the result of infection and sepsis, acquired in the community or complicating the course of a complex illness.

A general overview of the problems of sepsis and MODS, and the evolution of ideas in their description and management, can be found in the following references.

Bone R, Balk R, Cerra F, Dellinger RP, Fein A, Knaus W et al. : Definitions for sepsis and organ failure and guidelines for the use of innovative therapies in sepsis. Chest 1992; 101: 1644-1655.PMID 1303622

What is sepsis?Death for patients admitted to an intensive care unit is rarely a direct effect of the disorder that led to ICU admission. Rather, critically ill patients die as a consequence of a syndrome of secondary organ system disorders known as the multiple organ dysfunction syndrome (MODS). For the patient admitted with pneumonia, the infection is treated, but after a course characterised by renal failure, persistent hypotension, failure to wean from the ventilator, and recurrent bouts of nosocomial infection often with relatively low virulence organisms, the patient may succumb weeks later. A similar course follows admission for very divergent illnesses — acute pancreatitis, ruptured abdominal aortic aneurysm, refractory congestive heart failure, or decompensated liver cirrhosis.

The terminology conventionally used to describe the septic patient is confusing, and attempts to simplify or clarify these concepts really have not accomplished their objectives. This confusion arises from both the complexity of the disease, and our intellectual sloppiness in trying to describe it.

The word 'sepsis' is attributed to Hippocrates (460 – 370 BC). In Hippocrates´ view, living things died and were transformed through one of two basic, but opposite

processes. Pepsis denoted tissue breakdown that was restorative and life-giving, and was exemplified by the digestion of food, or the fermentation of grapes to produce wine. Sepsis, on the other hand, was tissue breakdown associated with disease, decay, a foul smell, and death – for example, putrefaction, infection of wounds, and the vapours arising from stagnant swamps. The advent of the microscope and the work of Louis Pasteur demonstrated that the processes of sepsis arose through the actions of micro-organisms; intriguingly, those of pepsis – whether fermentation or peptic ulceration – are also a product of microbial activity.

Sepsis to the clinician has a somewhat different connotation. It describes an acute clinical syndrome that is quite variable and often non-specific in its clinical presentation, but that suggests the presence of uncontrolled infection, and the threat of imminent clinical deterioration.

THINK How would you define sepsis? Must a patient have a culture-proven infection to be called septic, or is the essence of the syndrome the clinical response of the host? Do all patients with infection have sepsis? Do all patients with sepsis have infection?

Write down five physiological or laboratory abnormalities that you believe to be most characteristic of sepsis. Now ask a colleague to do the same, and compare your lists? Are they the same? Is there a single clinical syndrome that we might call sepsis?

The definition of sepsis is frustratingly elusive, because sepsis is not so much a syndrome or a disease, as it is a state of profoundly deranged host-microbial homeostasis. In health, humans, like all multicellular creatures, live in a state of symbiosis with the microbial world. We consume micro-organisms when we eat, they colonise our mucosal surfaces in health, and as mitochondria, they even power the fundamental processes of cellular metabolism that allow us to live. Bacteria in the gut aid in the digestion of food, stimulate intestinal epithelial development, and inhibit the growth of exogenous and potentially more virulent organisms. In turn, we provide nutrients and a favourable environment for growth, and a state of mutually beneficial tolerance ensues. In sepsis, however, this state is disrupted: micro-organisms become a threat to the host, and the host activates a series of processes to kill the micro-organism. Normal patterns of microbial colonisation are altered, as are normal patterns of the host antimicrobial response. Infection becomes both the cause of sepsis and a manifestation of the disorder; the host response, heralded by inflammation, becomes both evidence of the disorder, and a cause of the tissue destruction that results.

However, concepts such as this are perhaps too esoteric to guide clinical decision-making, and instead, we have, over time, developed a series of generally agreed upon, though imperfect, definitions for the clinical syndrome of sepsis.

Infection is a microbial phenomenon: the invasion of normally sterile host tissues by viable micro-organisms. Infection both triggers and exacerbates the host response, and should always be sought and treated. For example, patients with sterile pancreatic necrosis may have MODS but infected necrosis greatly increases the risk of death. Similarly, burn patients exhibit SIRS but most deaths are due to infection of the burn wound.

Microbial invasion typically evokes a response in the host that may be local, in which case the local signs of inflammation – rubor, calor, dolor, tumor, and functio laesa – are present,

or it may besystemic. Sepsis, therefore, is defined as the systemic host response to invasive infection.

However, it will be apparent that just as local signs of inflammation may arise from sterile insults – a foreign body, an allergic reaction, or an autoimmune disease such as arthritis, for example, so too systemic inflammation may result from a sterile insult – multiple trauma, burns, blood transfusion, a drug reaction, pancreatitis, or thrombotic thrombocytopenic purpura, to name a few. Thus the term thesystemic inflammatory response syndrome (SIRS) was coined to denote clinical manifestations of systemic inflammation, independent of cause; in this model, sepsis is present when SIRS arises as a consequence of infection from various sources, such as lung, abdomen, urinary tract, (solid blue circles in figure, below).

Overlap between infection, SIRS and sepsis

 

A moment´s thought will disclose a further nuance that must be considered. A systemic inflammatory response may be desirable and beneficial to the host when infection is present. Yet, clinicians generally use the word sepsis to denote a state that is perceived as a threat. Thus the severity of sepsis can be graded. Severe sepsis is sepsis in association with hypoperfusion and mediator-induced organ dysfunction, an intuitively undesirable state, while septic shock is severe sepsis of sufficient severity that perfusion is even more profoundly jeopardised, vaso-regulatory mechanisms being overcome and tissues becoming ischaemic. Some authors have proposed terms such as severe SIRS anddistributive or vasodilatory shock to describe the equivalent gradations in the patient without infection. Regardless of terminology, it is clear that each represents increasingly more profound derangements in systemic homeostasis, reflected in an increasing rate of mortality.

NOTEThe terminology of sepsis is inherently controversial and imprecise, but the underlying concepts are not.The acute clinical syndrome – however defined or characterised – identifies a patient at risk of clinical deterioration or death, and must prompt the clinician to institute adequate resuscitative measures rapidly. The syndrome reflects the response of the host, not the cause: it is critical that a diagnosis be established, appropriate therapy be instituted, and unnecessary interventions be minimised.

The immediate threat faced by the septic patient is not only the uncontrolled growth of bacteria, but also the consequences of the systemic inflammatory response on oxygen delivery to the tissues. In the most advanced stages, septic shock is said to be present – a state in which oxygen availability is compromised, normal oxidative metabolism at the

cellular level is jeopardised, and anaerobic metabolism occurs. This state is characterised biochemically by the release of lactate from the cells; although elevated serum lactate levels are neither sensitive nor specific for shock.

Altered cellular metabolism results in altered cellular function, and even in cell death as a consequence of necrosis or apoptosis. Apoptosis is a more physiologic and controlled process of programmed cell death, however, those programmes may be heightened due to inflammatory stimulation.

The biochemical processes of sepsis are enormously complex – the consequence of the coordinated interaction of literally hundreds of host-derived mediator molecules including cytokines, the complement and coagulation cascades, vasoactive mediators such as kinins, prostaglandins, acute phase reactants, and short-lived intermediates of oxygen and nitrogen. However, the resultant effects on blood flow and oxygen delivery to the tissue can be readily understood as the consequence of four key acute changes, described below.

Vasodilation

Vasodilation involving small arterioles and nutrient vessels reflects the presence of mediators and the dysfunction of compensatory mechanisms. A number of cytokines can induce the expression of an enzyme known as inducible nitric oxide synthase (iNOS) in vascular endothelial cells. iNOS catalyses the conversion of the amino acid arginine to citrulline, in the process generating a molecule of nitric oxide. Nitric oxide is a potent smooth muscle relaxing agent, and causes local vasodilatation –  indeed this property accounts for the pharmacologic effects of such classical vasodilatory agents as nitroglycerin and nitroprusside. This may be a local process that directly opposes sympathetic stimulation. Vasodilatation drastically increases the diameter of the vascular tree, reducing resistance, inducing relative hypovolaemia (as the volume required to fill the tree is significantly increased) and therefore lowers the effective blood pressure. Moreover, the loss of normal microvasculature resistance results in accelerating the passage of blood through the capillary beds, and therefore reduces the time available for passive unloading of oxygen from the saturated red.

Loss of endothelial barrier function

Loss of integrity of the endothelial barrier –  a consequence of disruption of the endothelial tight junctions and loss of endothelial cells –  results in loss of proteins and fluid into the interstitium. This further decreases the effective intravascular volume. Moreover, the resulting oedema aggravates cellular hypoxia by increasing the distance between the erythrocyte in the capillary, and the adjacent cells, and so increasing the distance that oxygen must diffuse to reach the cell.

Occlusion of capillaries

Occlusion of capillaries by thrombi, activated leukocytes, and aggregates of erythrocytes, whose capacity for deforming during passage through the microvasculature has been reduced, significantly impairs perfusion. Oxygenated blood bypasses these occluded capillaries (shunt), failing to unload the oxygen and therefore increasing the local tissue oxygen deficit.

Intravital sidestream dark field images of the sublingual microcirculation:capillaries with red cells flowing (normal, upper image) and obstructed with minimal flow (sepsis, lower image).

Images courtesy of C Ince, Amsterdam, the Netherlands

For further information, see the following references.

Impaired myocardial contractility

Impaired myocardial contractility as a consequence of poorly characterised myocardial depressant factors further affects compensation. Reduced myocardial contractility is readily demonstrable in animal models and in septic humans; its biologic basis is poorly understood and probably multifactorial. Moreover its significance is uncertain, since the cardiac output in sepsis is characteristically increased, and clinical evidence of impaired cardiac output commonly reflects inadequate fluid resuscitation.

The net consequence of these abnormalities is the classical haemodynamic profile of resuscitated sepsis: tachycardia, peripheral oedema, a hyperdynamic circulation (assuming adequate fluid resuscitation), warm extremities, and hypoperfusion characterised by elevated mixed venous oxygen saturation.

What is the pathologic basis for each of the following clinical features of severe sepsis or septic shock?

Tachycardia Tachypnoea Pyrexia Leukocytosis

A reduced systemic vascular resistance An increased mixed venous oxygen saturation

THINK The classical signs of local inflammation –  rubor, tumor, calor, dolor, and functio laesa – have their systemic counterparts in patients with systemic inflammation. What are they? Why is each of these beneficial to the host in the setting of local inflammation? Why is each detrimental in systemic inflammation?

Recognising sepsis in the critically ill patientEarlier consensus statements suggested that sepsis could be recognised by relatively non-specific physiologic alterations thought to be characteristic for SIRS and including tachycardia (a heart rate >90 beats per minute), tachypnoea (a respiratory rate >20 per minute or the need for mechanical ventilation), hypo- or hyperthermia (a core temperature <36.0 °C or >38.0 °C), and leukopenia or leukocytosis (a white blood cell count <4000/mm3 or >11 000/mm3). While each of these reflects the physiologic abnormalities that accompany the onset of systemic inflammation, they are neither specific for the process, nor comprehensive in encompassing the acute derangements that signal sepsis. What is more important from a clinical perspective is sensitivity to an acute change in clinical status that denotes the systemic derangements of sepsis. In fact a large number of acute changes in systemic physiologic homeostasis may be the early signs of the clinical syndrome of sepsis (see reference below).

Sepsis is not a disease, but a syndrome whose evolution should prompt the clinician to look for a treatable cause. The list of possible causes of SIRS is extensive (See Task 3   ). Infection is common, important, and generally readily treatable. For this reason it should always be considered first. However, the clinician must also consider such causes as tissue ischaemia, injury, non-infective inflammatory disorders such as pancreatitis or systemic lupus erythematosus, drug or transfusion reactions, and other immunologically-mediated processes such as thrombotic thrombocytopenic purpura. An educated team triage utilising simple tools or reminders of SIRS and possible infections will facilitate the constant vigilance required. If clinical monitoring systems are employed, simple alerts remind clinicians of potential inflammatory processes.

NOTEThe diagnosis of infection in a patient admitted from the community is usually straightforward, whereas establishing a definitive diagnosis of nosocomial infection may be much more challenging. As a general rule, early and appropriate treatment of community-acquired infection has a greater impact on outcome. For more information see the PACT module on Severe infection 

The early recognition of sepsis is not primarily the diagnosis of a specific disease, but the recognition that a patient is ill and in danger of acute deterioration, and that immediate intervention is needed to:

Restore haemodynamic stability and tissue perfusion Determine the cause, and reverse or correct it Institute appropriate physiologic support to prevent further tissue injury.

Let's now consider how these goals are accomplished.

2/ RESUSCITATION AND HAEMODYNAMIC SUPPORT OF THE SEPTIC PATIENT

The most immediate threat facing the septic patient is tissue hypoxia resulting from inadequate oxygen delivery (DO2) in the development and course of septic shock, defined by sepsis plus hypotension. At the cellular level, a lack of oxygen causes the cell to switch from aerobic to anaerobic metabolism. This shift is accompanied by increased production of lactate – the metabolic by-product of anaerobic metabolism – but also by altered cellular metabolism, or even cell death, because of inadequate energy generation.

Multiple factors contribute to tissue hypoxia in sepsis:

Peripheral vasodilatation resulting in an increased intravascular diameter requiring a higher vascular volume

Increased capillary permeability with leakage of protein-rich intravascular fluid into the interstitium

Maldistribution of blood flow at the capillary level – secondary to microvascular thrombosis, to capillary plugging by platelet thrombi or aggregates of red cells and leukocytes, or to altered vasomotor tone – resulting in shunting of oxygenated blood to the venous side of the circulation

Myocardial depression secondary to poorly characterised circulating factors.

How is oxygen carried in the blood transferred to cells adjacent to capillaries? List three factors that might impede cellular uptake of oxygen in sepsis.

Recognising the perfusion defect of sepsisImpaired tissue perfusion in sepsis in the development and course of septic shock is recognised through its physiologic consequences; the manifestations are many, though variable from one patient to the next.

Any acute change in normal haemodynamic homeostasis that is not readily explained by another obvious diagnosis (for example, haemorrhage, myocardial infarction, intoxications or poisoning) should prompt you to consider the possibility of septic shock, particularly when there is suspicion for infection.

Tachycardia, a common finding in sepsis, is in part, a reflexive response to a relative intravascular volume deficit. Physical examination of the patient with unresuscitated septic shock reveals signs of peripheral hypoperfusion, a rapid, weak pulse, cool extremities, and tissue pallor and cyanosis.

Reduced intravascular volume is also reflected in reduced pressure in the venous side of the circulation. Under normal circumstances, the right atrial or central venous pressure (CVP) is 0 to 5 mmHg, and the pulmonary capillary occlusion (or wedge) pressure (PAOP or PCWP) is 5 to 12 mmHg when the plateau of the cardiac function curve relating cardiac

output to filling pressure is approached; lower levels are highly suggestive of an intravascular volume deficit and of fluid responsiveness, i.e. a rise in cardiac output upon a fluid challenge. For more information see the PACT module on Haemodynamic monitoring   .

In the absence of factors that modify the distribution of blood in the microvasculature or the normal relationship between the capillaries and cells in the immediately adjacent tissues, reduced oxygen delivery results in increased oxygen extraction (VO2/DO2). This is a compensatory mechanism that maintains oxygen consumption, as blood moves more slowly through the microvasculature, and the gradient down which oxygen diffuses becomes steeper. When oxygen extraction has reached a maximum, further lowering of DO2 reduces VO2 below tissue needs. See the PACT module on Hypotension for more information   .

When oxygen extraction increases, the oxygen saturation of the haemoglobin of venous blood returning to the heart is lower than its normal 70% thus directly reflecting an increased release of oxygen and providing us with a measurement that indicates poor oxygen delivery. While the most representative sample of venous blood from the entire body is the mixed venous blood in the pulmonary artery (as it evaluates the mixing of total body including coronary sinus and cerebral blood), a sample drawn from the superior vena cava or right atrium through a central venous catheter provides a reasonably reliable estimate of the extent to which haemoglobin has become deoxygenated in the peripheral tissues. Low central venous saturations indicative of inadequate resuscitation were well demonstrated in early sepsis in the following study.

Following resuscitation, the central venous saturation rises for reasons discussed above. Decreased tissue perfusion can be detected indirectly as metabolic acidosis, or more directly, as an increased serum lactate level.

Reduced tissue perfusion results in acute alterations in organ function (see Task 5   ). This altered function is apparent in the kidney, where reduced blood flow results in reduced urine production (oliguria) and increasing serum creatinine (see the PACT module on Oliguria and anuria   ). It is also seen in the cardiovascular system as persistent tachycardia and hypotension (shock), and in the central nervous system, as confusion. Hepatic hypoperfusion can result in hepatocellular injury, reflected in elevated levels of hepatic transaminases. Pulmonary dysfunction is manifest as hypoxaemia due to ventilation–perfusion mismatching, despite increased oxygen support.

The clinical and biochemical manifestations of tissue hypoperfusion are, as a rule, readily apparent, and often multiple. A serialised measure of serum lactate in conjunction with central or mixed venous monitoring facilitates evaluation of benefits of treatment. Their rapid correction is critical.

Cardiovascular support in sepsisThe mainstay of cardiovascular support of the septic shock patient is the rapid administration of intravenous fluids with the goal of restoring haemodynamic stability. The choice of fluid is much less important than the amount. Large randomised trials, or systematic syntheses of smaller studies, have failed to provide convincing evidence for the superiority of crystalloids or colloids during fluid resuscitation.

Adequate intravenous access is established, and normal saline, Ringer´s lactate, or another balanced electrolyte solution is rapidly administered, with the goal of giving a litre or more of fluid within the first 30 minutes. Initial assessment of the response to resuscitation entails monitoring of vital signs and urine output, and so a urinary catheter provides the clinician with a tool to evaluate the dynamic response to intravenous fluids. Restoration of a normal blood pressure and heart rate, and maintenance of a urinary output greater than 0.5 ml/kg/hr (adults) suggest that initial resuscitative measures have been successful. But what if they aren't? More invasive strategies must be used to optimise tissue perfusion. The best validated of these, and the approach adopted by the Surviving Sepsis Campaign, is the algorithmic approach popularised by Rivers (below).

Treatment algorithm of septic shock

 

First, continue to provide intravenous fluids, but insert a central venous catheter to monitor the central venous pressure and the oxygen saturation of the central venous blood. Fluids

are given rapidly until the CVP is at least 6-8 mmHg, but continued improvement may occur as the CVP is pushed to even higher levels, and so it is advisable to continue fluid resuscitation as long as the blood pressure, urine output and, more specific measures of fluid responsiveness respond, and pulmonary gas exchange is not significantly jeopardised.

THINK Optimising fluid resuscitation requires maximising the benefits of further fluids, while minimising the risk –titrating therapy to the response of the individual patient.List three physiologic responses that suggest your patient is responding adequately.List three physiologic responses that suggest your patient is not responding appropriately, but may in fact be experiencing harm.

For the next six patients with central venous catheters in place, and to whom you are administering fluid, record the total amount of fluid you have given, the CVP level at which you decided to stop administering fluids, and the time it took to reach this goal from the time that resuscitation began. Are the responses similar in all patients? How do your actions compare with the recommendations of the Surviving Sepsis Campaign?

If the blood pressure remains low despite adequate volume resuscitation, then vasopressors should be administered, and dosed so that the mean arterial pressure is raised to about 65 mmHg. This target seeks to maximise vital organ perfusion, while minimising the adverse consequences of the resulting vasoconstriction. Either norepinephrine or dopamine are the vasopressor agents recommended by the Surviving Sepsis Campaign but there is no compelling evidence that one is superior to the other. Other vasoactive agents such as epinephrine (or perhaps vasopressin) might be equally effective. Practically speaking, it is best to use the agent whose dosing and side-effect profile you and your team are most comfortable with.

An increase in blood pressure does not necessarily result in increased tissue perfusion, and perfusion, not pressure, is our therapeutic objective. The adequacy of global tissue perfusion can be inferred by measuring the ScvO2, either with a specially designed catheter, or simply by blood gas analysis of a sample of blood withdrawn through the CVP catheter. The objective is to increase the ScvO2 to at least 70%, and this objective is achieved by one or both of two strategies. First, an inotropic agent such as dobutamine is administered at a dose of 5 µg/kg/min, and the dose titrated upwards based on the response of the central venous oxygen saturation. High doses of dobutamine have been associated with increased mortality, and so the total dose should probably not exceed 20 µg/kg/min. Second, if the haemoglobin level is low (less than 4.96 mmol/l; 8.0 g/dl), packed red cells can be transfused to raise the level to 6.2 mmol/l (10 g/dl).

Most patients will respond to these measures; those who do not pose a formidable challenge for the intensivist and the critical care team, and one for which approaches must be systematically clinical and empirical. Moreover their risk of death is high. A reasoned approach to this situation would consider diagnostic or iatrogenic error. First, re-evaluate the clinical situation. Is the failure to respond a function of refractory septic shock, or has another unrecognised complication occurred; for example, tension pneumothorax following insertion of a central catheter, severe pulmonary oedema resulting from aggressive

If the patient does not respond as expected, reconsider the possibility of another diagnosis

resuscitation, inappropriate/inadequate antimicrobial therapy or acute myocardial infarction with cardiogenic shock.

Once these are ruled out, consider the clinical context  is aggressive therapy likely to result in an acceptable quality of life for the patient, or are you simply engaging in a battle of wills with what is clearly an irreversible process? Is the therapeutic objective reasonable? Set your sights lower, and consider accepting a mean arterial pressure of 55, or even 50 mmHg, if trying to increase the pressure proves unsuccessful; consider a further fluid challenge if the CVP is high but oxygenation acceptable. Before you initiate a further intervention, define a therapeutic target, and decide what you must measure in order to evaluate whether you are achieving your objectives.

A 67-year-old man is admitted with septic shock following an extensive small bowel resection for intestinal infarction secondary to a superior mesenteric artery embolism. Following your initial resuscitation, he is tachycardic with a heart rate of 136/min, and hypotensive with a blood pressure of 76/40 mmHg; the central venous pressure is 12 mmHg, and the ScvO2 is 72%. He has produced 20 ml urine in the last two hours. His haemoglobin is 6.5 mmol/l (10.6 g/dl), and he is receiving norepinephrine at a dose of 20 µg/min. He is mechanically ventilated, and his SaO2 is 96% with an FiO2 of 0.6, and PEEP of 12 cmH2O.Describe three potential interventions you might employ to manage this man, and the specific parameters you will use to evaluate whether you have been successful.

3/ IDENTIFICATION AND CONTROL OF THE SOURCE OF INFECTION

Once resuscitation has been started, the next priority in the management of the septic patient is to establish a diagnosis and to institute appropriate measures to treat the inciting infection – specifically, performing appropriate cultures, thereafter starting empiric antibiotics directed against the probable infecting pathogen(s) and undertaking source control to eradicate a discrete infective focus.

Diagnosing infection in the acutely ill patientEstablishing an infective diagnosis in the acutely ill patient may be quite straightforward, but is often enormously challenging. Our objective is to answer, as reliably and rapidly as possible, three questions:

Is my patient infected? What is the site from which the infection is arising? What is/are the infecting pathogen(s)?

Is the patient infected?

The initial history and physical examination may establish a diagnosis of infection. In the patient presenting to the emergency department with an acute illness, fever, malaise, and a new cough productive of purulent sputum, this suggests the diagnosis of community-acquired pneumonia, while acute abdominal pain with findings of peritoneal irritation suggests intra-abdominal infection. Physical examination may provide other evidence of a localised infective process – a necrotising soft tissue infection, infected joint or head and

neck infection, for example. Recent hospital admission and indwelling catheters, among other examples should alert the clinician to other possible infections.

Laboratory investigations can support the presumptive diagnosis of infection. An increased white blood cell count is often present, however life-threatening infection can present with leukopenia. Although an increased percentage of immature forms should be present in either instance, the discriminatory ability of leukocyte changes to differentiate infective and non-infective causes of fever is poor. Some studies have shown increased levels of C-reactive protein to have a better discriminatory capacity, with an odds ratio of 5.4 for the diagnosis of infection in patients with clinical signs of systemic inflammation. Increased procalcitonin (PCT) levels provide even greater diagnostic accuracy, with an odds ratio of 15.7; procalcitonin has shown particular efficacy in discriminating infective from non-infective aetiologies in patients presenting to the emergency department with acute respiratory symptoms. Other biomarkers of infection – for example soluble TREM-1 for the diagnosis of pneumonia and endotoxin activity levels for the diagnosis of Gram-negative infection – show diagnostic promise but experience is limited.

Culture evidence of tissue invasion currently represents the gold standard for the diagnosis of infection. Specimens should be obtained prior to initiating antibiotics to maximise the diagnostic yield. Positive blood cultures provide objective evidence of systemic dissemination of a micro-organism; however the yield of blood culture positivity is variable, depending on the nature of the infection as well as the methods of sampling. Following adequate skin preparation, at least two samples should be taken from a peripheral site, inoculating at least 10 ml blood into each specimen bottle. Positive cultures, taken using sterile technique, of normally sterile tissue fluid – pleural or peritoneal fluid, for example – also provide conclusive evidence of infection. Cultures obtained from a surface are less reliable, for the differentiation of normal or pathologic colonisation from invasive infection is difficult. Positive sputum or urine cultures in intubated or catheterised patients may reflect colonisation of the tracheobronchial tree or lower urinary tract respectively; the use of quantitative culture techniques, and invasive interventions such as bronchoscopy to obtain specimens from a distal source, can improve the reliability of culture data.

Which of the following has not been shown to reduce the rate of contamination (false positivity) during the performance of blood cultures?a/ Skin preparation with chlorhexidine rather than povidone-iodineb/ Aspiration of blood through a central catheter using full sterile techniquec/ Changing the needle prior to inoculating the specimen into a culture bottled/ Disinfecting the stopper on the culture bottle prior to inoculation

Definitive identification of the isolated microbial species, and evaluation of its sensitivity profile to common antibiotics, typically involves a delay of at least two days. An earlier presumptive microbial diagnosis can be made using a Gram stain (particularly of CSF or urine samples), which can provide information on the class of organism within an hour.

When the suspected infection is a nosocomial infection arising in a hospitalised patient, the diagnosis is particularly challenging. For this reason, it is probably clinically more accurate to think of the probability that infection is present, rather than to try to definitively establish or rule out infection. In other words, a decision to treat with antibiotics is made not on the basis of a diagnosis of infection, but rather on the probability that infection is the cause of the presenting clinical syndrome. Therefore, after cultures have been collected, a broad-spectrum antibiotic should be considered, primarily based on the likely source of the infection and knowledge of local pathogens.

For the next ten patients you are called on to evaluate for suspected sepsis, write down your initial clinical impression of the probability of infection (very unlikely, unlikely, neutral, likely, very likely) and the presumptive infective focus. Repeat this evaluation three days later, using the results of laboratory, radiographic, and microbiological investigations to guide your reassessment. How accurate was your initial assessment? And don't be discouraged if it is imperfect. Diagnostic inaccuracy is one of the intrinsic challenges of managing the septic patient.

If the initial clinical assessment, augmented by the results of rapidly available investigations, suggests that infection is likely to be present, then empiric antibiotic therapy should be started expeditiously, guided by knowledge of local resistance patterns and the probable infective focus. Delays in initiating therapy when severe sepsis or septic shock is present have been associated with a time-dependent increase in the risk of death (figure below). Equally, escalation of antibiotic therapy should be considered if the patient fails to respond or if microbiological studies suggest this is necessary. Lastly, de-escalation must be appropriately invoked when culture results indicate that treatment with a narrower spectrum antibiotic is possible.

Delay between onset of hypotension and the initiation of antimicrobials versus mortality

From Kumar A, Roberts D, Wood KE, Light B, Parrillo JE, Shrama S et al. Duration of hypotension before initiation of effective antimicrobial therapy is the critical determinant of survival in human septic shock. Crit Care Med 2006; 34(6): 1589-1596. PMID 16625125

What is the site of the infection?

Determining the site from which the infection is arising enables the clinician to refine the prediction of the likely infecting organism(s), and to determine whether (surgical or interventional) source control measures are indicated for definitive management of the infective episode.

Physical examination may aid in establishing the origin of the infection, but radiologic imaging studies are typically needed to document the precise site of origin and extent of spread. A chest X-ray may support the diagnosis of pneumonia by showing lobar consolidation or other acute alterations. If an effusion is also present, particularly in the patient with pneumonia who has failed to respond to systemic antibiotics, the diagnosis of empyema should be entertained, and investigated by aspirating a sample of the pleural fluid.

Ultrasonography is inexpensive, and rapidly as well as widely available. Its greatest utility lies in detecting infections arising from an obstructed abdominal hollow viscus  –  for example, the gall bladder and biliary tree producing acute cholecystitis or cholangitis, respectively, or from the urinary tract in the case of pyelonephritis. Ultrasonography is operator-dependent, and its use may be limited by external dressings, or by significant quantities of air in the gastrointestinal tract.

Computerised tomography (CT) scanning is the most useful diagnostic modality for patients with deep space infections in the abdomen or thorax; it is also useful in evaluating the extent of complex soft tissue infections. Oral or rectal contrast aids in CT interpretation by delineating the lumen of the gastrointestinal tract, and by demonstrating leaks from the GI tract when these are present. Intravenous contrast permits the identification of major vascular structures, and can demonstrate areas of tissue non-perfusion, suggestive of ischaemia or infarction. For more information see the PACT module on Clinical imaging 

Accurate delineation of the anatomic site of infection provides a diagnosis of the infection responsible for the septic state, and may facilitate image-guided management using percutaneously placed drains, and therefore obviating the need for surgical intervention.

Fluid collections suggestive of abcesses, with drain in situ

Less common foci of infection should also be considered. In the patient with multiple positive blood cultures, a diagnosis of endocarditis should be considered, particularly in the patient with pre-existing risk factors –  for example, valvular heart disease or a prosthetic heart valve, intravenous drug abuse, a long-standing central venous catheter. The diagnosis is supported by clinical features and established by transoesophageal echocardiography and blood culture. Altered neurological status or focal findings suggests the possibility of a cerebral or spinal focus, and should be assessed further by CT scan or MRI. Invasive tests like bone marrow aspiration must be considered for systemic infections such as tuberculosis especially in the immunocompromised host.

What is the infecting pathogen?

Accurate identification of the nature and antibiotic sensitivity profile of the infecting pathogen enables the clinician to tailor antibiotic therapy, using a regimen that is effective, but narrow spectrum, and so minimally disruptive to the patient's endogenous flora.

An initial prediction of the likely infecting pathogen is made on the basis of the presumptive site of the infection (pneumonia, intra-abdominal infection, prosthetic device-related infection, etc), and on the mode of acquisition –  community-acquired or nosocomial. Presumptive empiric therapy can then be initiated with reasonable confidence that the actual infecting organism will be susceptible (see the PACT module on Severe infection   ).

Accurate identification of the infecting organism requires the results of microbiological culture and sensitivity testing, which typically are not available for two or three days after the initial specimens were taken. Once these data are available, the antibiotic spectrum should be narrowed to target the particular pathogen specifically; if cultures are negative after 72 hours, strong consideration should be given to stopping all antibiotics. Alternately, other causative organisms should be considered if the patient has persistent clinical signs of sepsis.

Managing infection in the septic patientOnce a presumptive diagnosis of infection as the cause of sepsis is made, treatment should be initiated expeditiously.

Systemic antibiotics appropriate to the working diagnosis and based on knowledge of local resistance patterns, should be given as soon as cultures have been drawn and, ideally, within an hour of presentation. Antibiotic selection is discussed further in the following reference and in the PACT module on Severe infection   .

When a discrete focus of infection is identified, source control measures should be considered, and implemented once the patient has been stabilised. Some general principles underlie their use.

A 74-year-old man is admitted to the ICU following repair of a ruptured abdominal aortic aneurysm; significant co-morbidities include hypertension, insulin-dependent diabetes mellitus, and chronic obstructive pulmonary disease. He is intubated, and has a Foley catheter, pulmonary artery catheter, peripheral intravenous, and nasogastric tube placed in the operating room (OR). What specific sites of nosocomial infection is he at risk for? Why?

What organisms are most likely to be cultured from each of these sites?

Source control is a term that encompasses all those physical measures undertaken to control a source of infection, to stop ongoing microbial contamination, and to restore optimal anatomy and function. Source control measures are applicable to most infections causing severe sepsis, and fall into two broad categories.

Drainage is the creation of a controlled sinus (a connection between a closed cavity and an epithelial surface) or fistula (an abnormal communication between two epithelially-lined surfaces). By draining an abscess (a localised focus of infection and host inflammatory cells, walled off by a fibrin capsule), a deep-seated infection is converted to a controlled sinus or a fistula (figure below).

Drainage creates a controlled sinus or fistula

Draining abcesses, percutaneously or surgicallyThe drain is a foreign body that keeps the tract open

Based on this principle, the optimal approach to achieving source control is that which accomplishes the objective with the least degree of upset to the patient, and with full consideration to minimising the risk associated with subsequent reconstructive efforts. For example, perforated diverticulitis causing severe sepsis can be managed operatively by resection of the involved segment of colon, followed by the creation of an end-colostomy, or by primary anastomosis. Resection removes the site of ongoing contamination, while the colostomy or anastomosis is the controlled fistula (because it is an abnormal communication between two epithelially-lined surfaces). However, if the CT scan shows a localised process, percutaneous image-guided drainage can accomplish the same objectives with a lesser degree of physiologic insult to the patient. Drainage procedures are most effective if the infected material is liquid; if it includes solid tissue, then debridement must be employed.

Debridement is the mechanical removal of infected non-viable tissue, typically by surgical resection, but in the case of superficial infection, by the use of dressing changes, or in the case of an infected foreign body, by its removal. As a general rule, debridement of infected necrotic tissue should be performed as rapidly as feasible: for patients with necrotising soft tissue infection, for example, prognosis is directly related to the interval between onset and definitive surgery. However, the benefits of debridement must be weighed against the risks of intervention: for patients with infected peripancreatic necrosis, it has become evident that delayed debridement is preferable, because of the substantial risk of uncontrollable

haemorrhage that may occur when adequate demarcation of viable and non-viable tissues has not occurred.

The ultimate objective in managing the patient with severe sepsis or septic shock is to achieve survival with a good quality of life. Careful consideration of the definitive measures needed to manage a focus of infection can shorten the period of time the patient is hospitalised, and improve the ultimate quality of life. For example, avoiding a stoma will obviate the need for subsequent surgical reconstruction and its attendant risks. When a stoma is deemed necessary, creating it in such a manner that it can be closed through a single small incision will reduce the morbidity associated with subsequent reconstruction.

THINK For each of the following clinical scenarios resulting in severe sepsis or septic shock, consider:

What are the options for achieving source control? How can the problem be managed using the principles outlined above? What is your preferred approach and why?

A 66-year-old woman presents to the emergency department with fever, chills, jaundice, and right upper quadrant pain, 14 years after a cholecystectomy. Her blood pressure is 70/40, and her heart rate 130/minute. An ultrasound exam shows the common bile duct to be dilated to 1.8 mm.

A 47-year-old man presents with diffuse abdominal pain; he is hypotensive and tachypnoeic, with a temperature of 38.7 °C. Following resuscitation, he undergoes CT scanning which shows sigmoid diverticulitis with an associated abscess.

An 86-year-old woman develops confusion, a supraventricular dysrhythmia, and oliguria, three days following a right hemicolectomy for cancer. She is afebrile. Her CT scan is shown below.

CT scan with free air in the abdomen suggestive of leaking anastomosis

A 59-year-old man, intubated five days after a right lower lobectomy for lung cancer, develops a fever, and a new right-sided infiltrate. There is an associated pleural effusion; diagnostic thoracentesis yields Gram-positive cocci.

THINK Invasive devices are a common nidus of infection in critically ill patients. What factors contribute to device colonisation?What steps can be taken to reduce the risk of colonisation?For which of the following nosocomial infections should device removal be an integral part of infection management, and under what circumstances?

Bacteraemia Urinary tract infection Ventilator-associated pneumonia Septic thrombophlebitis Peritonitis during chronic ambulatory peritoneal dialysis A-V or V-V shunts for chronic renal replacement therapy Indwelling central catheters (short or long-term)

Non-infective causes of severe SIRSA diagnosis of severe sepsis or septic shock implies an infective aetiology. However an identical clinical syndrome may evolve in the absence of infection, and management principles do not differ, with the important exception that anti-infective measures are unnecessary, and potentially even harmful.

To establish that infection is not the cause of the acute illness requires primarily the exclusion of infection as a cause, and secondarily making a reasonable alternative diagnosis (table below). Appropriate empiric antibiotics and a systematic attempt to establish an anatomic and microbiologic diagnosis are indicated if there is a moderate or high probability of an infective cause. However, it is equally important, if these investigations fail to confirm a diagnosis of infection, that empiric antibiotics be stopped as soon as the diagnosis has been ruled out, and certainly by 72 hours after initial presentation.

Injury Multiple, Trauma, VILI, HgeIschemia -reperfusion Limb ischemia, opening the abd in

compartmental syndrome, rupture Aorta aneurysm

Inflammation sterile Acute pancreatitis Immune SLE, GVHt, Transplant rejection,

collagenicIdiopathic Toxic epidermal , wegners, TTPIndicrine DKA, Addisons, HyperthyrodismIntoxication ASAIatrogenic Transfusion , drugs

4/ ADJUNCTIVE THERAPY FOR SEPSIS

Despite advances in our understanding of sepsis, the mortality after treatment is over 50% in the presence of shock. Earlier in this module, the importance of diagnosis, early resuscitation and directed therapy for sepsis were emphasised. It is, however, crucial to appreciate that several other aspects of the disease and its treatment also impact on the outcome from sepsis. This discussion will highlight the adjunctive interventions, based on available evidence, which may be considered part of a comprehensive approach to treatment. Such interventions must equally be with minimal risk of adverse events. The considerations for paediatric patients are uniquely different and will not be addressed.

Several therapeutic interventions have been investigated for sepsis in an attempt to reduce morbidity and mortality. Where the benefits of such therapy are unproven, their introduction cannot be justified.

Adjunctive therapy for sepsis

*rhAPC: recombinant human activated protein C,DVT: deep vein thrombosis,LOS: length of stay,NOS: Nitric oxide synthase

Intervention Endpoint SERh APC Dose related antiinflammatory, anticoagulant, 

profibrinolyticBleeding

Steroids  Dose related reduction NOSGlucose control Normoglycemia HypoglycemiaBlood products Increase oxygen delivery InfectionSedation , analgesia, ms relaxants

Reduce O2 consumption Increase length of stay, polyneropathy

DVT prophylaxis BleedingStress ulcer prophylaxsis

Prvent stress ulcer Increase nosocomial pneumonia

Recombinant human activated protein C

Many novel therapeutic agents have been investigated over the last three decades. None have shown incontrovertible evidence of benefit in patients with sepsis. Recently, recombinant human activated protein C (rhAPC) has been shown to reduce mortality (absolute mortality reduction of

6.1%) in adult patients with severe sepsis when administered to patients within 24 hours of diagnosis of severe sepsis and sepsis induced organ dysfunction. There are many postulated mechanisms of action including reduction of thrombin formation, an anti-inflammatory effect and reduction of rolling of monocytes and neutrophils. The dose required is 24 µg/kg/hour for 96 hours. Bleeding is the commonest complication associated with therapy. Treatment with rhAPC should be avoided in patients with active bleeding or at high risk for bleeding and the drug has not yet been shown to be effective in less severe septic patients.

Treatment must be curtailed where significant risks are associated with drug administration.

The Surviving Sepsis Campaign guidelines (2008) recommend that the drug be considered in septic patients with an APACHE 2 score ≥25 but not in those with a score <20.

NOTEThe European Medicines Agency (EMeA) and other regulatory bodies have individualised the indications for rhAPC use in severe sepsis and it is advised that you know the licensed indication in your own jurisdiction.http://www.emea.europa.eu/htms/human/epar/a.htm

In patients who require surgery, the drug should be discontinued for at least one hour prior to surgery. It may be recommenced one hour after minor procedures and 12 hours after major surgery – once concern re bleeding complications has subsided.

Absolute contraindications for rhAPC Active internal bleeding Recent (within 3 months) haemorrhagic stroke Recent (within 2 months) intracranial or intraspinal surgery, or severe head

trauma Trauma with an increased risk of life-threatening bleeding Presence of an epidural catheter Intracranial neoplasm or mass lesion or evidence of cerebral herniation Thrombocytopenia <30 x 109/l.

Steroids

Earlier studies using high dose steroid therapy for sepsis demonstrated no mortality benefit and were associated with an increased prevalence of nosocomial sepsis. Low dose hydrocortisone (200-300 mg per day) was shown in the Annane (single-centre) study to reduce mortality in corticotropin unresponsive patients with septic shock who require fluids and vasopressor support. However, the larger, multi-centre CORTICUS study showed only that hydrocortisone hastened the reversal of shock in those in whom shock was reversed. Hydrocortisone did not improve survival or reversal of shock in patients with septic shock, either overall or in patients who did not have a response to corticotropin.

When using hydrocortisone, treatment should not be delayed for confirmation of the presence of hypoadrenalism (by corticotropin stimulation testing).

The mechanism of action is thought to be via improving catecholamine receptor responsiveness to exogenous catecholamines. Modulating immune responsiveness in patients who have relative adrenal insufficiency is also thought to occur. Treatment with steroids should be weaned once shock has resolved. It is common practice to administer steroids for 5-7 days as abrupt withdrawal of therapy may result in adverse haemodynamic and immunological effects. There are no data supporting the use of steroids for protracted periods and a higher rate of steroid-related septic complications have been confirmed, even in the 11 day duration of therapy utilised in the CORTICUS study (above).

NOTETherapeutic interventions must only be introduced when there is clear evidence of benefit AND where the risks of treatment are out-weighed by the benefits.

THINK A known asthmatic is being ventilated for severe bronchospasm. He develops septic shock from a nosocomial pneumonia. What are the considerations for steroid therapy in this patient? The following scenarios illustrate the multiple reasons to start steroids in this, and many other cases.Three scenarios are possible in this case:

Steroids prior to admission: If the patient is a known asthmatic, it is possible that he would have been receiving steroids prior to ICU admission. In this instance, he would be on steroids at the time of development of nosocomial sepsis, in which case steroid therapy must be continued to treat acute on chronic bronchospasm and must be continued after the episode of septic shock has resolved.

Steroids for bronchospasm: The patient may not have been on steroids prior to admission in which case steroids may have been administered for the management of refractory bronchospasm. In this instance, steroid therapy must be withdrawn after resolution of bronchospasm or seven days after treatment for septic shock, whichever is later.

Steroids for septic shock: The standard approach to steroid therapy for fluid resuscitated and vasopressor-dependent septic shock should be considered.

A patient has been treated for an episode of septic shock with steroids. Two days later, he develops another episode of septic shock. Would you administer steroids again? Justify your answer.

Glucose control

Glucose control was demonstrated to reduce morbidity and mortality in surgical patients when the serum glucose was kept between 4.4-5.5 mmol/l (80-100 mg/dl). In medical

patients there is a significant reduction in morbidity, however, the benefits were more significantly realised in medical patients with an ICU stay >3 days. Glucose control should be initiated as soon as initial stabilisation of the patient has been completed. It is recommended that the glucose level to be targeted is <8.3 mmol/l (<150 mg/dl), with a goal of 4.4-5.5 mmol/l (80 to 100 mg/dl). This higher level is suggested because of the significant risk of hypoglycaemia. Glycogen depletion during sepsis increases the risk of hypoglycaemia. Hypoglycaemia has been reported even during the conduct of rigorous clinical trials. This is a serious life-threatening complication in the sedated patient and must be assiduously avoided.

Clear adherence to a well thought through protocol, well-trained nursing/medical involvement and supervision, regular blood testing and routine administration of glucose containing solutions (to avoid inadvertent hypoglycaemia) must be considered mandatory when patients are receiving insulin. Hourly glucose testing should be performed until the levels have stabilised and thereafter four hourly testing is acceptable. Point-of-care testing has been shown to overestimate glucose levels and must be used with caution.

THINK What are the clinical signs of hypoglycaemia? How would you suspect and diagnose hypoglycaemia in a sedated, ventilated patient?

Blood products

Red blood cells

Current data suggest that a haemoglobin level between 7-9 g/dl is as effective as a level between 10-12 g/dl in resuscitated patients. The Rivers trial mentioned above indicates this may not be true in sepsis patients with shock and persistent lactic acidosis or low ScvO2 on initial presentation. Red blood cells (RBCs) increase the oxygen carrying capacity of blood but do not immediately release bound oxygen and therefore may not result in increases in oxygen consumption. In addition, increasing the RBC concentration may adversely affect blood rheology thereby reducing oxygen delivery. Since the ideal haemoglobin concentration is unknown, RBC transfusion should likely be directed to achieving a level of 7-9 g/dl. In certain groups of patients where oxygen delivery and consumption is critical e.g. post-cardiac surgery or persistent ischaemic lactic acidosis, higher levels of haemoglobin (10-12 g/dl) may be preferable.

Platelets and fresh frozen plasma

Platelets should be administered when the count is <5 x 109/l since these patients are at risk for spontaneous bleeding. A platelet count >50 x 109/l is recommended prior to surgery. Abnormal laboratory tests of coagulation alone should not be used as an indication for coagulation factors. Fresh frozen plasma and/or platelets should be used in the presence of documented bleeding or prior to surgery and abnormal laboratory tests suggest their use. For more information see the PACT module on Bleeding and thrombosis 

Erythropoietin and antithrombin

There are no data to support the use of either therapy in patients with sepsis. Erythropoietin may be indicated for other conditions e.g. pre-existing chronic renal failure and impaired erythropoietin production. High dose antithrombin increases bleeding when co-administered with heparin.

THINK A patient with intra-abdominal sepsis receiving rhAPC requires major surgery. The drug infusion is stopped one hour prior to surgery. Excessive blood loss with anaemia occurs in ICU after surgery. What causes of bleeding should be considered and what should be done?

Analgesia, sedation and muscle relaxation

Patients with sepsis frequently require mechanical ventilation. Details on airway care, intubation and extubation, ventilation, analgesia, sedation and the use of muscle relaxants are covered in the PACT modules on Airway management   , Mechanical ventilation   and Sedation   .

THINK The ICU nurse reports that a patient with sepsis requires increasing doses of sedative drugs without achieving adequate sedation. What are the likely causes for this behaviour?

Thromboembolism prophylaxis

ICU patients are at risk for the development of deep vein thrombosis (DVT) and pulmonary embolus (PE). Prophylaxis has been shown to reduce the prevalence of these complications, which should therefore be considered to be standard care for patients with sepsis. Unfractionated heparin (UFH) (eight hourly) is as efficacious as low molecular weight heparin (LMWH) (once daily) when administered subcutaneously. UFH should be used instead of LMWH when cost constraints are a factor. LMWH is renally excreted and may accumulate in patients with severe renal dysfunction. A dosage reduction/monitoring of anti-xa has been recommended in patients with creatinine clearance of <30 ml/minute.

In patients with active bleeding, coagulation disturbances and proven heparin-induced thrombocytopenia (HIT), both agents should be avoided and anticoagulation with non-heparin agents may be considered. Mechanical devices should be used in such patients. Patients at high risk for DVT and PE should receive both drug prophylaxis (UFH or LMWH) and mechanical devices (compression stockings or intermittent compression devices).

THINK What factors constitute high risk for DVT/PE? What are the clinical signs of PE?

For more information see the PACT module on Bleeding and thrombosis   .

Stress ulcer prophylaxis

Upper gastrointestinal bleeds occur less frequently in ICU patients who receive prophylaxis for stress ulcers but there is no evidence of a reduction in mortality from this therapy. Given that mechanical ventilation, coagulopathy and hypotension are the risk factors and that these occur commonly in patients with sepsis, prophylaxis should be administered in all

patients with severe sepsis. Histamine-2 (H2) receptor antagonists and proton pump inhibitors are equivalent in producing this benefit. The choice of agent should be based on cost, availability of drug and side effects associated with these agents. Sucralfate has been shown to be less effective than H2 receptor antagonists. For more information see the PACT modules on Nutrition   and Infection control strategies   .

Intravenous immunoglobulin

Several studies have suggested the use of intravenous immunoglobulin (IVIG) as an adjunctive intervention in patients with sepsis. The rationale is based on augmentation of the immune response to infection. The Surviving Sepsis Campaign Guidelines suggest that immunoglobulin may be considered in children with severe sepsis but there is no corresponding suggestion of recommendation in adults. There have been three recent meta-analyses but given differences in study methodology, issues relating to dose standardisation and safety, not to mention other advances in the management of sepsis, a large, prospective randomised trial is indicated before this form of treatment should be considered as routine adjunctive therapy.

Granulocyte colony-stimulating factor

Granulocyte colony-stimulating factor (G-CSF) has been used in neutropenic patients for several years to bolster receptor-mediated neutrophil and macrophage responsiveness and has been used clinically in severely septic, neutropenic neonates. Serum concentrations of G-CSF are elevated in sepsis but have no relationship to outcome. The Surviving Sepsis Campaign Guidelines do not include mention of G-CSF and further studies are required to elucidate where this intervention may be beneficial.

In conclusion, adjunctive therapeutic strategies have reduced morbidity and mortality associated with sepsis and should be routinely considered in clinical practice. These interventions should be carefully evaluated for benefit to ensure that unacceptable side effects (e.g. hypoglycaemia, bleeding or superinfection are controlled as far as possible. An area that will require careful ongoing consideration is the potential negative consequences from the multiple therapies that have been advocated for seps

5/ MINIMISING ORGAN DYSFUNCTION IN ICU

Acute multiple organ dysfunction is the defining complication of severe sepsis and septic shock. Organ dysfunction arises as a consequence of the profound systemic homeostatic derangements that accompany an uncontrolled systemic inflammatory response. It also develops, however, as a consequence of what the healthcare team does to treat these derangements. Organ dysfunction therefore may develop because of both the successes and the shortcomings of contemporary ICU care. Whilst all organ systems are affected, the cardiac and respiratory dysfunction typically manifest early in the clinical course of severe sepsis.

What is MODS?MODS can be defined as the development of acute physiologic organ system dysfunction following an acute life-threatening insult. Its manifestations may involve virtually every aspect of normal physiologic function but by convention, the syndrome of MODS is usually

considered to involve derangements in the function of six organ systems – the respiratory, cardiovascular, renal, haematologic, hepatic, and neurologic systems. The concept that the syndrome represents dysfunction rather than failure is preferred, since the process is reversible, and prognosis correlates not only with the number of failing systems, but also with the degree of dysfunction within a system. This conceptual model is reflected in the emergence of a number of similar systems to quantify the severity of the syndrome using a score (Table below).

Measuring Organ Dysfunction: the MOD and sequential organ failure assessment (SOFA) scores

 

The capacity to quantify MODS provides the clinician with a tool to measure changes in global severity of illness over time, and the investigator with a means of evaluating the epidemiology and natural history of critical illness. The MODS score uses six continuous physiologic variables, and so measures organ dysfunction independent of a therapeutic decision, since such decisions vary and may not only reflect evolving illness, but contribute to its evolution. Function of the cardiovascular system is measured using a simple calculated variable, analogous to the PO2/FiO2 ratio – the pressure-adjusted rate (PAR), calculated as the product of the heart rate and CVP, divided by the mean arterial pressure, and reflecting hypotension that does not respond to fluid resuscitation:

A normal value is 10 or less; increasing values reflect worsening cardiovascular function. In the absence of an actual measure of right atrial pressure, a normal value of 8 is used. The SOFA score incorporates therapy-dependent variables in the measurement of respiratory and cardiovascular dysfunction; urine output is also included as a measure of renal function. These differences, however, have only a minimal effect on the reliability of each.

The measuring systems provide different and complementary information on the clinical course of the patient. The score on the day of admission is a measure of illness severity at the onset of care, and correlates directly with ultimate prognosis: it is, in effect, a snapshot of how sick the patient is before critical care begins. Scores measured on a daily basis provide a reflection of how the global course of illness is evolving. Moreover the domains of improvement or deterioration can be followed over time, particularly if the raw data for each variable are used – is respiratory function, measured by the PaO2/FiO2 ratio, getting better or worse? Is there particular deterioration or improvement in one organ system that might

point to a correctable problem? Change over time can also be measured using the delta score – either by taking the worst values in each system over the ICU stay, and subtracting them from the admission score, or by looking at the change in scores from one day to the next; the former calculation provides a global measure of new organ dysfunction developing during the ICU stay, and so is a reflection of potentially preventable ICU morbidity.

Preventing organ dysfunction in the critically illThe ultimate objective of critical care practice is to support a patient during a period of life-threatening physiologic instability, while treating the process that triggered the illness on the one hand and minimising the associated multi-organ dysfunction including the adverse consequences of support measures. For convenience, this topic can be addressed in terms of one organ at a time.

Respiratory dysfunction

The acute lung injury (ALI) and acute respiratory distress syndrome (ARDS) reflect a particularly severe form of the respiratory dysfunction that develops in many critically ill patients. This respiratory dysfunction results from both the systemic inflammatory response to the acute injury that led to ICU admission, and the local response of the lung to additional, and preventable insults arising from the technique of ventilatory support used, or superimposed lung infection producing ventilator-associated pneumonia. A state of generalised vasodilatation and increased capillary permeability results in interstitial oedema in the lung, and exudation of protein-rich fluid into the alveolar space. Both of these compromise gas exchange, producing alterations that can be detected early in the course of the process as tachypnoea, hypoxaemia, and hypocarbia, as well as loss of lung compliance. The institution of mechanical ventilation introduces a new insult to the vulnerable lung, as positive pressure ventilation applies unnatural shear forces to the lung units, and repeated opening and closing of these units causes local tissue injury.

This injury evokes a further inflammatory response, with the influx of neutrophils into the lung, further exacerbating the local injury, and compromising gas exchange. Thrombosis in small, medium, or large vessels – secondary either to systemic activation of the coagulation cascade, or to embolisation from the extremities, further compromises gas exchange. Aspiration of bacteria from the stomach, oropharynx, and endotracheal tube evokes a further response, and adds to the complex picture of ALI. Finally, the process of tissue repair, with local fibrosis, contains the injured tissue, but at the cost of its function, and gas exchange is even further impaired. While the initial insult has usually already occurred by the time the physician sees the patient, the subsequent secondary injury is eminently amenable to preventive measures.

Gas exchange in the lung is compromised by alveolar oedema, and so early pulmonary function can be improved by judicious fluid resuscitation, administering a volume adequate to support the circulatory system without producing excessive alveolar or interstitial oedema. Monitoring resuscitation through the measurement of central venous pressure can aid in attaining this objective. It remains controversial whether clinically relevant oedema can be further reduced by the use of colloids such as albumin or starch solution in preference to crystalloids.

The development of ventilator-associated pneumonia (VAP) exacerbates acute respiratory dysfunction. A variety of strategies have been shown to be effective in reducing the risk of

VAP, including elevation of the head of the bed, use of closed system suction devices, reduced frequency of ventilator tubing changes, and subglottic drainage of secretions.

Selective decontamination of the digestive tract (SDD) – a strategy that reduces pathologic colonisation of the oropharynx and GI tract through the use of topical non-absorbed antibiotics and a short course of a parenteral agent – has been shown to be very effective in reducing the risk of VAP. Similarly, avoiding over-sedation, and removing the endotracheal tube as early as is feasible are additional effective strategies to lower the risk of VAP.

Protective lung ventilation strategies, accomplished by the use of low tidal volumes (6-8 ml/kg) and higher levels of PEEP, have been shown to reduce not only pulmonary, but also distant organ injury, and to improve survival. These effects may be modulated by the gradual introduction of pressure control ventilation and appropriate dosing of sedation. For more information see the PACT modules on Mechanical ventilation   , Respiratory failure   , Respiratory monitoring   and Haemodynamic monitoring   .

Cardiovascular dysfunction

The cardiovascular dysfunction of MODS includes peripheral vasodilatation, increased capillary permeability, microvascular occlusion with deranged capillary flow, and myocardial depression. While these abnormalities are not readily manipulated directly, the ultimate consequence – tissue hypoxia – can be attenuated by judicious clinical management.

The adverse effects of tissue oedema can be minimised. The use of a goal-directed approach, dosing fluid volumes to physiologic response can aid in refining fluid resuscitation. Commonly, once resuscitation has been accomplished, the patient will remain in a significantly positive fluid balance for many more days; cautious removal of this excess volume using pharmacologically-induced diuresis or renal replacement therapies can aid in mobilising this fluid.

The objective of haemodynamic support is to maximise oxygen delivery to the tissues (DO2). Since tissue oxygenation cannot be readily measured directly, it must be inferred indirectly. A reduced venous saturation indicates increased O2 extraction, and so suggests tissue hypoxia. Similarly an elevated lactate level suggests anaerobic metabolism secondary to reduced DO2. Both of these abnormalities have multiple causes, and therefore must be interpreted carefully. Hypotension is also inferred to reflect impaired DO2 since one must assume either inadequate cardiac output or loss of vascular tone, or both. However increasing blood pressure pharmacologically may not increase DO2, but actually reduce it, since increased pressure is achieved through constriction of, and perhaps reduction of flow in, small vessels. Other effects of circulating mediators may decrease the red cell ability to deform and flow through the capillaries, consumption of clotting products may cause micro-occlusion and all of these types of events may actually promote a further shunting of oxygenated blood, therefore an increase in ScvO2 in the face of increasing clinical dysfunction and lactic acidosis. For more information see the PACT module on Hypotension   .

Optimal cardiovascular management to minimise new organ dysfunction is complex, often counter-intuitive, and not at all well-defined. Beyond adequate resuscitation as outlined in our second task   , there is no evidence that invasive monitoring with a pulmonary artery catheter, attempts to drive haemodynamic parameters to supranormal values, or aggressive transfusion of RBCs can improve outcome.

Renal dysfunction

Renal dysfunction in the critically ill patient most commonly results from hypoperfusion, and so early, aggressive, and adequate resuscitation is the key to minimising subsequent renal dysfunction. In the patient with intra-abdominal pathology – for example, severe necrotising pancreatitis, a ruptured abdominal aortic aneurysm, or multiple abdominal trauma – renal perfusion may be impaired following resuscitation as a consequence of the development of an abdominal compartment syndrome. The diagnosis is established by the measurement of bladder pressure as a reflection of intra-abdominal pressure. Finally, nephrotoxic medications can contribute to the evolution of acute tubular dysfunction and acute kidney injury (AKI). For more information see the PACT modules on Acute renal failure  and Abdominal problems 

Hepatic dysfunction

The causes of evolving hepatic dysfunction in the critically ill are multifactorial, however several modifiable causes deserve particular attention. Ischaemia – evidenced as biochemical evidence of hepatocellular and reflected in significant rises in hepatic transaminases – is the most common cause of early hepatic dysfunction; hyperbilirubinemia is usually absent, or of very modest degree. Total parenteral nutrition can induce hepatocellular cholestasis; enteral nutrition is preferred if possible. Massive transfusion may cause jaundice. Finally, hepatotoxic medications may produce a picture of evolving cholestasis. Once liver failure has occurred, the lactate levels will become even less specific to tissue perfusion. For more information see the PACT module on Acute hepatic failure 

Haematologic dysfunction

Early haematologic dysfunction in the critically ill may be caused by depletion of coagulation factors as a consequence of massive bleeding. Alternatively, such dysfunction may occur as a result of intravascular consumption during the process of disseminated intravascular coagulation (DIC). Special attention should be paid to the inverse relationships between consumptive coagulopathy and tissue hypoperfusion. In both cases, prophylaxis is targeted at the cause. Thrombocytopenia may also reflect the development of heparin-induced thrombocytopenia (HIT). Discontinuation of heparin exposure is therapeutic. For more information see the PACT module on Bleeding and thrombosis 

Neurologic dysfunction

Although abnormalities in peripheral nerve function have been described in the critically ill, the most obvious and clinically most important manifestation of altered neurologic function is a reduced level of consciousness. Excessive sedation is an important contributing factor, and the ICU stay can be shortened, and outcomes improved, by a strategy of deliberate daily wakening of the critically ill patient. Drowsiness, agitation, or confusion may also reflect the side effects of other medications, the sequelae of aggressive fluid resuscitation with subclinical cerebral oedema, or the effects of disruption of normal patterns of sleep and day/night cycling.

While corticosteroids may improve haemodynamic function early in the course of septic shock, their use has been associated with muscle weakness during convalescence from critical illness. For more information see the PACT module on Neuromuscular conditions 

Nosocomial infection contributing to organ dysfunction

Optimal management of the critically ill patient necessitates a careful titration of support to maximise benefit and minimise harm.

Nosocomial infection in the critically ill may arise from exogenous reservoirs, but more typically results from infection with endogenous organisms whose emergence reflects a combination of altered patterns of colonisation, and impairment of physical and other host defence barriers.

Invasive devices such as central catheters, endotracheal tubes, and urinary catheters are particularly susceptible to colonisation by micro-organisms that produce biofilms e.g. coagulase-negative staphylococci. The colonised device both bypasses normal physical barriers, and serves as a reservoir for continuous microbial seeding. Successful treatment involves removal of the device when feasible; prophylactic measures include adequate sterile technique during device insertion, and reducing unnecessary usage.

The predominance of antibiotic-resistant organisms in nosocomial ICU-acquired infection also reflects the important role played by antibiotic pressures in selecting for resistant species. Judicious use of antimicrobial agents based on the use of narrow spectrum, culture-guided therapy, shorter duration of treatment, and discontinuation of empiric antibiotics in the face of negative cultures are all important strategies to reduce antimicrobial pressures on microbial ecology.

Randomised controlled trials and systematic syntheses of the data from these provide strong evidence that both rates of nosocomial infection, and ICU mortality can be reduced through the use of SDD. SDD entails the topical application of non-absorbed antibiotics with broad spectrum activity against Gram-negative aerobes (polymyxin B and tobramycin) and fungi (amphotericin), supplemented by a 3 to 4 day course of a systemic agent such as cefotaxime. The regimen preserves the anaerobic flora of the gastrointestinal tract, and has no significant activity against Gram-positive organisms. Despite extensive evidence of efficacy, and despite the lack of any compelling evidence of harm secondary to the emergence of resistant organisms, SDD is not widely used

Treating organ dysfunction in the critically ill

A comprehensive overview of the support of the critically ill patient with organ dysfunction would comprise a textbook of critical care. Specific considerations in the support of individual failing organs are covered in other modules of PACT. For our purposes here, we will focus on a few basic principles in the provision of such support.

Organ system support does not cure the underlying disease: it merely sustains life while the clinician identifies a treatable process, or sometimes, while the patient gets better despite the absence of a compelling diagnosis. The degree of organ system support is a reflection of the gravity of the clinical situation, and mortality increases with both the number of failing organ systems, and the degree of dysfunction within each. The goal of therapy is to liberate the patient from exogenous support.

With this objective in mind, it is important to acknowledge that patients do not require the support we provide; rather we choose (for clinically compelling reasons) to provide that support, and equally, good clinical care entails careful consideration of how important that support is to a successful outcome. If the mean arterial pressure is 60, therefore, it is

important to ask the question, 'Do I need to increase it to improve tissue oxygenation?', rather than simply to institute vasoactive therapy to increase it to an arbitrary level such as 65 or 70. The patient has no intrinsic need for vasopressors; rather you have a therapeutic decision to make, whether artificially increasing the mean arterial pressure will improve tissue oxygenation, and so increase the probability of survival. Often, as in this case, there are little data from clinical trials to guide that decision: good clinical judgment and careful evaluation of the consequences of a decision must guide practice.

Similarly, patient care is best served by an explicit attempt to liberate patients from the need for exogenous support as rapidly as is safe and feasible. While there may be something comforting about a sedated, haemodynamically stable patient receiving full mechanical ventilation, explicit attempts to wean the patient by daily spontaneous breathing trials are associated with a superior clinical outcome.

Finally, organ dysfunction is the clinical consequence of a pathologic process, and faced with new or evolving organ dysfunction, every effort should be made to identify a treatable cause. Does new onset organ dysfunction herald an unrecognised complication of recent surgery, or an adverse effect of a newly prescribed medication? Does it reflect the adverse sequelae of another intervention such as mechanical ventilation, transfusion, or total parenteral nutrition? Is it the initial presentation of a nosocomial infection? The desired clinical trajectory for the critically ill patient is one of improving physiologic function and liberation from exogenous support. Deviations from this course must be managed, but more importantly, they must be viewed as possible clinical evidence of a new complication.

Further information on organ dysfunction and support can be found in PACT modules on Haemodynamic monitoring   , Acute renal failure   , Oliguria and anuria   , Acute hepatic failure   , Bleeding and thrombosis   , Severe infection

Organ dysfunction and clinical futility

Although increasing degrees of organ dysfunction portend a worse clinical outcome, organ dysfunction alone is not the factor that guides a clinician to suggest termination of aggressive support. Organ dysfunction is a dynamic process and the critical factor in determining its reversibility is its cause and its evolution over time. The prognosis for ultimate survival is reduced when organ dysfunction fails to resolve over an extended period of time, or when organ dysfunction worsens in the absence of a treatable cause. However decisions regarding the appropriateness of continuing life sustaining therapy are individualised and grounded in considerations of the presence of treatable disease, the influence of co-morbid conditions, and patient preferences for life prolonging support. Consideration of the degree or trajectory of organ dysfunction can support, but not determine these decisions.

CONCLUSION

Sepsis and SIRS are common causes of critical illness, MODS and of morbidity and mortality. The key to a successful approach to clinical management is timely recognition of sepsis/SIRS, early resuscitative treatment, concurrent diagnosis and specific therapy including surgical/interventional source control where indicated. Adjunctive therapies can improve the clinical course and survival. Organ supportive therapy is the mainstay of critical care but needs to be titrated carefully to achieve maximum benefit while minimising any

associated adverse effects. Multidisciplinary collaboration is important to clinical management.

PATIENT CHALLENGES

A 67-year-old male presents with a two-day history of chills, high fever, cough and shortness of breath. He is known to have diabetes mellitus type II, mild hypertension and mild COPD. On admission to the emergency department (ED) the patient appears ill, is coughing, restless and sleepy, but rousable. Body temperature is 40.2 °C, blood pressure 80/40 mmHg, heart rate 120 b/min. (regular), respiratory rate 26/min.

On physical examination crackles are present over the right hemithorax and a stat chest X-ray (figure) shows a consolidation mainly in the right middle and lower lobes.

NOTERecognition of the life-threatening nature of this condition and the need of prompt adequate treatment is mandatory. It may save his life.

Laboratory results show:

A metabolic acidosis (pH 7.21, HCO3- 16 mmol/l)

Hypoxaemia (PaO2 58 mmHg [7.7 kPa], pCO2 25 mmHg [3.3 kPa], SaO2 89% on 2 l O2 by nasal cannula)

Haemoglobin 6.8 mmol/l (10.9 g/dl) Leukocytes 16.4 x 109/l (80% granulocytes) Platelets 60 x 109/l Creatinine 160 µmol/l (1.8 mg/dl) Urea 14 mmol/l (39.2 mg/dl) Bilirubin 10 µmol/l (0.58 mg/dl) Liver enzymes slightly elevated Lactate 4.3 mmol/l (38.7 mg/dl) Glucose 14 mmol/l (252.3 mg/dl)

What is the diagnosis? How urgent is treatment?

Learning issues  

Recognition of severe sepsis and septic shock

What is your initial treatment?

Refresh your knowledge on treatment modalities of septic shock. This will guide the choice of your therapy.

You are pursuing a two-pronged approach to the severe sepsis/septic shock involving optimisation of tissue perfusion and specific treatment of the cause of the infection/sepsis. When cultures have provided the identification of the causative organism(s) and their sensitivity/resistance pattern, antibiotic therapy can be adjusted to these findings. Septic shock, with its concurrent generalised tissue hypoperfusion, is the major cause of MODS in the critical care situation. At this time point there is already dysfunction of several organ systems.

Learning issues  

  Identification of clinical situations prone to develop MODS

  Strategies to prevent MODS or further aggravation of organ dysfunction

Specify some of these organ dysfunctions.

Is rapid attention to reversing tissue hypoxia/hypoperfusion important to the final outcome?

Learning issues  

Reversing tissue hypoxia/hypoperfusion

 

How can unintended, additional deterioration of organ dysfunction in your patient be prevented?

NOTEBe constantly aware of the possibility of iatrogenic injury and the risk of nosocomial infection.

After initial resuscitation in the ED, the patient is admitted to the ICU. In the following hours the patient becomes haemodynamically stable after fluid treatment and institution of norepinephrine (0.4 µg/kg/min). Blood oxygenation, however, further deteriorates in the hours following admission and the patient is intubated and mechanically ventilated (TV 6 ml/kg, PEEP 8 cmH2O, FiO2 0.5) with improvement in blood gases (PaO2 70 mmHg [9.3 kPa], SaO2 95%). 

An initial Gram stain of a tracheal aspirate shows an abundance of Gram-positive diplococci and cultures of both sputum and blood confirm two days later the presence of penicillin-sensitive pneumococci. The initially instituted broad-spectrum antibiotic regimen is narrowed down to high-dose penicillin i.v. on day two.

Learning issues  

Management of MODS in septic patients

NOTEAll septic shock patients should be treated in ICU where possible.

Learning issues  

  Using laboratory data to narrow the antimicrobial spectrum of therapy

  PACT module on Severe infection

How do you manage the hyperglycaemia in your patient?

Is there evidential support for tight glucose control in this clinical scenario? Are there cautions required?

Learning issues  

Blood glucose control

 

On the day of admission, insertion of a pulmonary artery catheter is considered but at this time not done since after resuscitation, the patient remains haemodynamically stable and starts to diurese (40 ml/hour). Moreover, metabolic acidosis gradually improves and 12 hours after the start of treatment the blood lactate level has fallen to 2.8 mmol/l. Over the next two days the patient further improves, temperature declines and there is a slight increase in PaO2/FiO2 ratio. Also, renal clearance improves as evidenced by a decreasing serum creatinine and platelet count increases to 85 x 109/l.

What do you do now to minimise the risk of iatrogenic exacerbation of organ dysfunction?

NOTEInsertion of a pulmonary artery catheter or other invasive monitoring devices are not routine procedures in septic shock. They may be

useful options however, when haemodynamic instability persists with an adequate CVP, or when ARDS and renal impairment develops. Careful clinical observation and monitoring of intervention targets are the cornerstones of management.

  PACT module on Haemodynamic monitoring

On day four after admission, however, the patient becomes haemodynamically unstable and the vasopressor dose has to be increased (dopamine 15 µg/kg/min, norepinephrine 0.6 µg/kg/min with a CVP of 13 mmHg) to attain a MAP of 70 mmHg . Body temperature rises again above 38.5 °C, and shows peaks of 40 °C, platelet count falls to 45 x 109/l and oliguria worsens.

Apparently something is wrong with your patient now. What is your strategy?

Think logically and systematically

  Causes of haemodynamic instability in septic patients with MODS

What do you consider a likely cause of the haemodynamic instability in this patient and how do you evaluate for this?

NOTESeveral causes can be simultaneously operative

  PACT module on Haemodynamic monitoring

Is this deterioration likely to be septic in nature?

How do you approach the diagnosis of this new episode of likely sepsis?

  Nosocomial infection  

  PACT module on Severe infection

Is adrenal dysfunction a possibility? What do you do?

  Adrenal dysfunction in sepsis  

  PACT module on Homeostasis

  PACT module on Hypotension

  Hydrocortisone in septic shock

With increasing vasopressor therapy you have obtained a MAP of 70 mmHg which you consider acceptable and you decide to insert a pulmonary artery catheter which was uneventful. Measurements reveal a hyperdynamic circulation (CI 4.8 l/min/m2 and SVRI 900 dynes/sec/cm–5/m2) and a wedge pressure (PAOP) of 16 mmHg. This virtually rules out hypovolaemia and cardiac output seems adequate. You give 100 mg hydrocortisone i.v., to be repeated after 8 and 16 hours. However, urine output is still low (20 ml/hr) and serum creatinine has risen to 180 µmol/l (2.0 mg/dl).

Learning issues  

Prevention of acute renal failure in sepsis

 

What can you do to prevent further deterioration of renal function and acute kidney injury (AKI)?

NOTEPrevention of acute renal failure is of utmost importance since mortality in sepsis markedly increases

when ARF complicates its course.

NOTEIn sepsis renal blood flow becomes highly dependent on mean arterial pressure. The precise mean arterial pressure target is difficult to predict but is typically >80 mmHg

What is the nature of the renal dysfunction and how would you manage the norepinephrine therapy?

NOTEKeep in mind that mechanisms other than hypoperfusion may affect renal function in sepsis. This should not discourage you from optimising renal perfusion pressure.

You know from textbooks that prerenal dysfunction can be assessed by measuring urinary sodium concentration and osmolality. Lab results show a urine osmolality of 700 mosmol/l and urine sodium concentration of 15 mmol/l.

NOTEThese tests have a poor specificity and sensitivity and have not been validated in critically ill patients.

Learning issues  

PACT module on Oliguria and anuria

 

What is your interpretation of these results?

NOTEThe kidney remains vulnerable to episodes of hypotension also in the recovery phase.

Do you reduce the dose of penicillin now that the plasma creatinine level has increased to 180 µmol/l (2.0 mg/dl)? Explain your answer.

NOTEAlways consider potential adverse effects of treatment on renal (and other organ) function.

You increase the dose of norepinephrine which results in a MAP of 85 mmHg without a major change in cardiac output. Diuresis starts to increase slightly (35 ml/hr). You feel that you have now optimised haemodynamics in your patient. In the meantime you are addressing another real clinical problem: the rise in body temperature. The approach that may be used can be helpful in all situations where new signs or symptoms appear.

How do you approach the diagnosis of the new fever?

Learning issues  

Causes and evaluation of fever in critically ill patients  

PACT module on Pyrexia

 

What are the possible 'missed diagnoses' in this patient?

NOTEIn this septic MODS patient recurrence of fever is a sign of (an exacerbation of) an inflammatory process unless another cause is

established. This could significantly contribute to further deterioration of organ function and therefore needs urgent diagnosis and treatment. Radiologic studies have a high priority here since they are rapidly obtained and may reveal (or rule out) a diagnosis.

CXR shows resolution of the pulmonary infiltrates but there is an increased opacity at the right side compatible with pleural fluid as shown in the figure on the left. In retrospect, daily chest X-rays show the gradual development of pleural fluid partly obscured by dense infiltrates. A sinus X-ray series shows no opacities.

What is your next step following this finding?

The diagnostic pleural puncture reveals yellow-stained turbid fluid. A Gram stain shows many polymorphonuclear leukocytes and sparsely some Gram-positive cocci. 

Chemistry: protein 20 g/l, pH 7.10 (blood 7.42), LDH 1150/U/l (blood 210/U/l), glucose 3.8 mmol/l (blood 6.4 mmol/l).

 

What is your conclusion and what should be done?

  PACT module on Communication skills

You make preparations to insert a drainage tube. The patient has thrombocytopenia (45 x 109/l) but only mildly increased clotting times. The nurse asks you whether platelet concentrate should be ordered.

 

How do you respond to the nurse?

Learning issues  

Invasive procedures in thrombocytopenia PACT module on Bleeding and thrombosis

A chest tube is inserted and 1.5 l turbid fluid drained from the right pleural space which is flushed twice daily with sterile normal saline. After chest tube drainage and treatment with hydrocortisone, on day six the patient becomes haemodynamically stable allowing for weaning from vasopressor support. Prophylactic subcutaneous low molecular weight heparin (LMWH) is continued as prophylaxis against thromboembolism.

 

What could be the cause of the increasing thrombocytopenia on day four in this patient?

Learning issues  

Thrombocytopenia in sepsis

PACT module on Bleeding and thrombosis

 

NOTEFailure of platelet counts to rise during treatment of sepsis should raise suspicions of missed septic foci ('missed diagnosis') or inadequate (antibiotic) treatment.

Learning issues  

Disseminated coagulopathy in sepsis

Would you consider a platelet transfusion in this patient? Explain your answer.

  PACT module on Bleeding and thrombosis

NOTEThe best treatment of sepsis-induced thrombocytopenia is adequate treatment of the septic process.

What pharmacological measures can you consider in order to prevent further renal injury in your patient?

Learning issues  

PACT module on Oliguria and anuria  

PACT module on Acute renal failure

NOTEWhen potentially nephrotoxic drugs cannot be avoided, plasma levels are monitored to ensure efficacy of the agent and to minimise toxicity.

In the following days your patient further improves. Body temperature normalises, platelet count trends upward and a short polyuric phase (without frusemide) precedes a normalisation of renal function. Cultures of blood and pleural fluid taken on day four show no growth. 

In the meantime, the patient is successfully weaned from mechanical ventilation and haemodynamic support including hydrocortisone. The chest tube is removed after seven days and a CXR shows only a residual thickening of the right pleura. The patient is discharged from the ICU 12 days after admission in a relatively good condition.

Learning issues  

Possible role of frusemide (furosemide) PACT module on Oliguria and anuria

A 53-year-old man with a history of non-insulin-dependent diabetes (type II) underwent a right hemicolectomy for cancer four days ago. You are called to the surgical floor to evaluate him because of confusion and new onset of generalised abdominal pain. His blood pressure is 70/40 mmHg, his heart rate 136 bpm, respiratory rate 28/min, and temperature 38.6 °C. A Foley (urinary) catheter is inserted, and drains a small amount of concentrated urine; a transcutaneous oxygen saturation monitor records an SaO2 of 98%.

Learning issues  

 Recognising sepsis in the critically ill patient

 Cardiovascular support

What are your immediate priorities?

  PACT module on Haemodynamic monitoring

As the fluid is being administered, you insert a subclavian line, and record a central venous pressure of 2 mmHg; the ScvO2 is 52%. You check the blood glucose patient record and it is normal.

What is the most likely diagnosis and the differential diagnosis?

Learning issues  

 Diagnosing infection in the acutely ill patient

 PACT module on Abdominal problems

Because of this, you continue resuscitative measures and you draw blood for culture, and start systemic antibiotic therapy with a carbapenem.

Learning issues  

 Managing infection in the septic patient  

  Instituting antibiotic therapy  

  PACT module on Severe infection

What is his pressure-adjusted rate (PAR) at this time?

 This value is within normal limits, and, in conjunction with a low MAP, suggest a functional intravascular volume deficit that may respond to fluids.

NOTEPending knowledge of the offending organism(s), you should know your local hospital formulary recommendations for new onset abdominal sepsis and for the appropriate escalation of therapy when earlier antibiotic therapy has already been in place.

Learning issues  

PAR   –     a derived index for circulatory assessment

By the time the patient reaches ICU, 3.5 litres of crystalloid have been infused and the heart rate has dropped to 100, his blood pressure has increased to 110/80 mmHg and his CVP is now 9 mmHg.

What would his PAR now be and how do you interpret the result?

Despite continued resuscitation and a total of 5 litres of crystalloids, he becomes tachycardic again at 120/min and hypotensive with a blood pressure of 80/50, and the urine output has been 30 ml urine over the past hour. The central venous pressure has now increased to 12, and the SaO2 on supplemental oxygen by mask is 93%. The ScvO2 is 58%.

What are your priorities in the ICU?

Because of a falling SaO2, a reduced level of consciousness, and a need for ongoing resuscitation, you elect to intubate him, and initiate mechanical ventilation. Blood work is sent, and an abdominal CT arranged. He is now on an FiO2 of 0.6, with 12.5 cmH2O of PEEP and the arterial blood gas shows a PaO2 of 80 mmHg (10.6 kPa). Blood work shows:

Haemoglobin of 8.7 mmol/l (14.0 g/dl) White count of 18.4 Platelet count of 140 000 Lactate of 4.2 mmol/l (37.8 mg/dl) Blood sugar of 18.6 mmol/l (335 mg/dl) Creatinine of 229 µmol/l (2.5 mg/dl) Bilirubin of 21 µmol/l (1.2 mg/dl).

Stabilisation appears not to be possible until the cause of his acute deterioration has been addressed.

Learning issues  

PACT module on Mechanical ventilation

What are the priorities in terms of 'source control'?

What is his MOD score at this time?

He is transferred to the operating room, and at laparotomy is found to have a dehiscence of his anastomosis, with diffuse purulent peritonitis. The abdomen is irrigated, and the disrupted anastomosis exteriorised as an ileostomy and a mucous fistula; the abdomen is closed primarily. He receives a further 6.5 litres of fluid intraoperatively, and is maintained on an epinephrine infusion. 

Over the next six hours following his transfer back to the ICU, his urine output decreases, his creatinine rises to 279 µmol/l (3.1 mg/dl), and his tidal volume decreases. The PIP (and pPlat) continue to increase and the ventilator is constantly alarming. The dose of dobutamine and norepinephrine has increased, and his haemodynamic parameters reveal a blood pressure of 80/50, heart rate 140, CVP of 22 mmHg, and ScvO2 of 62. His PAR at this time is now over 30.

Learning issues  

Source control

How do you interpret his deterioration? What should be done?

Learning issues  

 PACT module on Oliguria and anuria    PACT module on Abdominal problems

The Foley catheter is transduced, and the estimated intra-abdominal pressure is 42 mmHg. He is taken back to the operating room, and the incision reopened, achieving temporary closure with an absorbable mesh. His cardiorespiratory status improves. 

It is now the morning of the next day. He remains on norepinephrine and dobutamine, but the ScVO2 is now 74%, and he is on an FiO2 of 0.5, with a PaO2 of 80 mmHg (10.6 kPa). HR is 102 and BP is now stable at 110/74.

Learning issues  

Intra-abdominal pressurePACT module on Abdominal problems

Are there adjunctive therapies that should be considered at this stage?

Learning issues  

Adjunctive therapy for sepsis

Because he continues to remain febrile, vasopressor dependent, and his platelet count is dropping, but is not low enough to contraindicate protein C, activated protein C (aPC) is started later on.

Are there other (supportive) measures that you maintain and review at this stage?

  PACT module on Nutrition

  PACT module on Abdominal problems

  http://www.survivingsepsis.org/

He remains intubated and sedated over the next five days. His MOD score has only fallen marginally to 11, having been 14 at admission. He continues to have a low grade fever on antibiotics; the organisms from his peritoneal pus (taken at surgery) are covered by the current antimicrobial therapy and there is no clinical evidence of nosocomial pneumonia and sputum cultures are sterile.

What else would you do?

The CT scan confirms the presence of a large pelvic abscess. Judicious exploration of the wound at the bedside by the surgeon and yourself allows entry to the collection, and several hundred ml of foul-smelling purulent material is drained. Had this not been possible, you would have opted for percutaneous drainage. Following drainage of this pelvic abscess, the patient improves. He is weaned from vasoactive support, and a tracheostomy is performed. The platelet count rises. Urine output increases, and as he begins to mobilise fluid, his level of consciousness improves. He is weaned from the ventilator during the fourth ICU week, and discharged to the ward two days later.

Learning issues  

Managing infection   –   drainage

Learning issues  

 PACT module on Airway management

Learning issues  

PACT module on Mechanical ventilation

On reflection, rapid diagnosis and treatment is essential in the clinical management of sepsis. Both cases illustrate the dual approach involving concurrent resuscitation/organ support and specific therapy targeted to the source of the infection/sepsis. Careful clinical evaluation is required for diagnostic precision from the outset and to allow ongoing vigilance to prevent secondary or recurrent problems which are frequent in patients with sepsis.

Q1. Management of septic shock due to colonic anastomotic breakdown includes  Your answers

A. Prompt administration of appropriate antibiotics

The correct answer is : True

B. Targeting an ScvO2 of 60% or more

The correct answer is : False

C. Repeat laparotomy if the intra-abdominal pressure measured via a Foley (bladder) catheter equals 15 mmHg

The correct answer is : False

D. Immediate administration of activated protein C

The correct answer is : False

E. Low-dose hydrocortisone in those with fluid- and vasopressor-resistant hypotension

The correct answer is : True

Your total score is 0/5

Q2. In an adult patient, the following are clinical features of the early course of severe sepsisQ3. The following are mechanisms responsible for tissue hypoxia in severe sepsis  Your answers

A. Relative hypovolaemia causing hypotension

The correct answer is : True

B. Maldistribution of blood flow because of microvascular thrombosis

The correct answer is : True

C. Maldistribution of blood flow because of arterio-venous shunting

The correct answer is : True

D. Loss of intravascular fluid due to capillary leakage

The correct answer is : True

E. Myocardial depression due to coronary vasospasm

The correct answer is : False

Your total score is 0/5

Q4. Optimal timing for initial interventions after recognition of septic shock is  Your answers

A. Within one hour for intravenous corticosteroids

The correct answer is : False - Corticosteroids in this setting are an adjunctive therapy; they are recommended if vasopressors are needed after adequate fluid resuscitation. Corticosteroids are not a resuscitative intervention.

B. Within one hour to obtain adequate cultures

The correct answer is : True

C. Within six hours for aggressive fluid resuscitation

The correct answer is : False - Fluid resuscitation of a patient in septic shock should begin immediately after recognition of this clinical situation

D. Within 12 hours for vasopressors after fluid resuscitation

The correct answer is : False - Vasopressors, to restore adequate blood pressure, should be started whenever needed, as soon as possible, to complement fluid resuscitation

E. Within one hour for intravenous antibiotic therapy against the likely pathogen

The correct answer is : True - Effective antimicrobial therapy should be started within one hour: delay between start of hypotension and antibiotics is a determinant of

survival

Your total score is 0/5

Q5. A 64-year-old man has been treated for necrotising pancreatitis with septic shock and multiorgan dysfunction including acute renal failure. It takes three weeks of treatment before stabilisation is achieved and weaning from mechanical ventilation can be considered. Two days after stopping all sedation the patient remains drowsy and unresponsive to command. He is on pressure support ventilation with adequate oxygenation and ventilation. Tone and peripheral reflexes are normal. The preferred strategy in this condition is  Your answers

A. Performing a CT-scan of the brain

The correct answer is : False - CT not indicated as there are no lateralising signs and septic/drug-related encephalopathy is the most likely

B. Ordering an EMG

The correct answer is : False - Not indicated as tone and reflexes are normal and other explanation (Answer A above) most likely

C. Expectant

The correct answer is : True - Reasonable to allow time for septic encephalopathy to resolve and elimination of drug effect to occur especially in light of the renal failure which can impair opiate (especially morphine metabolite) elimination

D. Starting an infusion of naloxone

The correct answer is : False - Not indicated, unless given very cautiously in the diagnosis of possible excess opiate effect. Normally benefit of such intervention would not outweigh the possible adverse effect

E. Starting an infusion of flumazenil

The correct answer is : False - Not indicated, unless given very cautiously in the diagnosis of possible excess benzodiazepine effect. Normally benefit of such intervention would not outweigh the possible adverse effect

Your total score is 0/5

top related